What single resistor could replace 20 Ω, 30 Ω and 50 Ω resistors which have been joined in series ?

Answers

Answer 1

Answer:

sine the resistors are connected in series, we could add the sum and that single value replaces the individual resistors in series. therefore, 100ohm resistor can do.


Related Questions

Which tool would you choose to create a storyboard, and why?

Answers

Answer:

Canva. Although Canva's free storyboard app for iOS and Android is targeted more at video production, it still provides animators and UX designers everything they need.

Explanation:

What is the speed of a helicopter that traveled 1500 miles in 7 hours?

Answers

the answer is....

214.28
which would be 214.3 or 214

An elf pushes a sleigh with force of 9N over a distance of 11m. How much work did the elf. Do on the sleigh

Answers

Answer:

99 J

Explanation:

The work done by an object can be found by using the formula

workdone = force × distance

From the question we have

workdone = 9 × 11

We have the final answer as

99 J

Hope this helps you

What is the relationship between a planet’s distance from the sun and the length of its year? Explain.

Answers

Answer:

Planets that are close to the sun have shorter years, and planets that are farther have longer years, because of how fast/slow it orbits the sun.

Explanation:

I WILL GIVE YOU BRIANLIEST PLEASE HELP!!!!!!
A small plane starts from rest and accelerates uniformly to the east to a takeoff velocity of 70 m/s in 5 seconds. What is the plane’s acceleration?

Answers

hope this helps, if you need clarification i got u

What are electromagnetic waves?

Answers

Answer:

Electromagnetic waves or EM waves are waves that are created as a result of vibrations between an electric field and a magnetic field. In other words, EM waves are composed of oscillating magnetic and electric fields. They are also perpendicular to the direction of the EM

Explanation:

What is the minimum number of 20-amp 277-volt branch circuits required where 810 incandescent luminaires are to be installed

Answers

Complete Question

What is the minimum number of 20-amp 277-volt branch circuits required where 810 incandescent luminaries are to be installed?The  lamps to be installed are rated 75 watts at 277 volts. This is NOT a continuous load so each circuit can carry 20 amps maximum

Answer:

The number is  [tex]k = 11[/tex]

Explanation:

From the question we are told that

     The current is  [tex]I = 20 \ A[/tex]

     The voltage is  [tex]V = 277 \ volt[/tex]

     The power rating of each bulb is  [tex]P = 75 \ W[/tex]

      The total number of  incandescent luminaries is [tex]n = 810[/tex]

Generally the amount of current each lamp takes is mathematically represented as

       [tex]I_e = \frac{P}{ V}[/tex]

=>    [tex]I_e = \frac{75}{ 277}[/tex]

=>    [tex]I_e = 0.270 \ A[/tex]

Generally the number of bulb fitting(i.e bulbs )  a  20 A current can supply (i.e the bulbs the branch ) to is

                    [tex]N = \frac{I}{I_e}[/tex]

=>                 [tex]N = \frac{20 }{ 0.270 }[/tex]

=>                 [tex]N = 73.86[/tex]

Generally the number of 20-amp 277-volt branch circuits required is mathematically represented as

          [tex]k = \frac{n}{N}[/tex]

=>       [tex]k = \frac{810}{73.86}[/tex]

=>       [tex]k = 11[/tex]

     

   

A rocket takes off against the force of gravity. Consider this a non-isolated system. Derive the rocket equation formula relating velocity to the mass of the rocket and time. Let the velocity of the rocket be v(t) and the relative speedof the ejected fuelrelative to the moving rocket be u. Ignore air drag.

Answers

Answer:

Explanation:

The solution of the question has been put in attachment.

To initiate a nuclear reaction, an experimental nuclear physicist wants to shoot a proton into a 5.50-fm-diameter 12C nucleus. The proton must impact the nucleus with a kinetic energy of 2.30 MeV. Assume the nucleus remains at rest. With what speed must the proton be fired toward the target?

Answers

Answer:

[tex]V_1= 3.4*10^7m/s[/tex]

Explanation:

From the question we are told that

Nucleus diameter [tex]d=5.50-fm[/tex]

a 12C nucleus

Required kinetic energy [tex]K=2.30 MeV[/tex]

Generally initial speed of proton must be determined,applying the law of conservation of energy we have

            [tex]K_2 +U_2=K_1+U_1[/tex]

where

[tex]K_1[/tex] =initial kinetic energy

[tex]K_2[/tex] =final kinetic energy

[tex]U_1[/tex] =initial electric potential

[tex]U_2[/tex] =final electric potential

mathematically

   [tex]U_2 = \frac{Kq_pq_c}{r_2}[/tex]

where

[tex]r_f[/tex]=distance b/w charges

[tex]q_c[/tex]=nucleus charge [tex]=6(1.6*10^-^1^9C)[/tex]

[tex]K[/tex]=constant

[tex]q_p[/tex]=proton charge

Generally kinetic energy is know as

         [tex]K=\frac{1}{2} mv^2[/tex]

Therefore

         [tex]U_2 = \frac{Kq_pq_c}{r_2} + K_2=\frac{1}{2} mv_1^2 +U_1[/tex]

Generally equation for radius is [tex]d/2[/tex]

Mathematically solving for radius of nucleus

         [tex]R=(\frac{5.50}{2}) (\frac{1*10^-^1^5m}{1fm})[/tex]

         [tex]R=2.75*10^-^1^5m[/tex]

Generally we can easily solving mathematically substitute into v_1

   [tex]q_p[/tex][tex]=6(1.6*10^-^1^9C)[/tex]

   [tex]K_1=9.0*10^9 N-m^2/C^2[/tex]

   [tex]U_1= 0[/tex]

   [tex]R=2.75*10^-^1^5m[/tex]

   [tex]K=2.30 MeV[/tex]

   [tex]m= 1.67*10^-^2^7kg[/tex]

   [tex]V_1= (\frac{2}{1.67*10^-^2^7kg})^1^/^2 (\frac{(9.0*10^9 N-m^2/C^2)*(6(1.6*10^-^1^9C)(1.6*10^-^1^9C)}{2.75*10^-^1^5m+2.30 MeV(\frac{1.6*10^-^1^3 J}{1 MeV}) }[/tex]

    [tex]V_1= 3.4*10^7m/s[/tex]

Therefore the proton must be fired out with a speed of [tex]V_1= 3.4*10^7m/s[/tex]

A solid cube of aluminum (density of 2.7 g/cm³) has a volume of 0.9 cm³. how many atoms are contained in the cube?​

Answers

Answer:

[tex]0.542*10^{23}\ Aluminum\ Atoms[/tex]

Explanation:

[tex]We\ are\ given:\\Density\ of\ aluminum=2.7\ g/cm^3\\Volume\ of\ aluminum-cube=0.9\ cm^3\\Hence,\\As\ we\ know\ that,\\Density=\frac{Mass}{Volume}\\Mass=Density*Volume\\Hence,\ here,\\Mass\ of\ the\ solid\ iron\ cube=2.7*0.9=2.43\ g\\Now,\\We\ also\ know\ that,\\Gram\ Atomic\ mass\ of\ Aluminum = 26.98 \approx 27\ g\\Hence,\\No.\ of\ particles=\frac{Mass}{GAM}*Avagadro's Constant\\Hence,\ here\\No.\ of\ Aluminum\ atoms=\frac{2.43}{27}*6.022*10^{23} \approx 0.542*10^{23}\ Aluminum\ Atoms[/tex]

Help me! Btw he’s playing golf.

Answers

Answer:

Muscular energy

Explanation:

Hope it helps!!!!

An object has more momentum if it has which of the following?

Answers

Answer:

C.) A high velocity and Large mass.

Explanation:

Momentum of any object is defined by following formula

Here :  m = mass of object

v = velocity of object

now we know that since momentum is product of mass and velocity

So in order to have more momentum we need the value of this product to be more.  So this product will me large is both the physical quantity will be more in magnitude. So if mass is large and velocity will be more then the product of them will be large and hence the momentum of object will be more. Btw I had that question too.

Which of these quantities impact centripetal force?
A.mass and speed only
B.mass and radius only
C.speed and radius only
D.mass, speed, and radius all impact centripetal force

Answers

Answer: D) Mass, speed and radius all impact the centripetal force

Explanation:

If we change either the mass, speed, and/or radius, then the centripetal force will be different. Though on rare occasions we could change the variables together in a way to have the centripetal forces line up, but in general the before and after forces will likely be different. Changing one variable at a time only will definitely result in a different centripetal force.

This is because the centripetal force F is determined by the formula

F = (m*v^2)/r

where,

m = mass

v = velocity

r = radius

A softball player swings a bat, accelerating it from rest to 2.2 rev/srev/s in a time of 0.20 ss . Approximate the bat as a 2.3-kgkg uniform rod of length 0.96 mm , and compute the torque the player applies to one end of it.

Answers

Answer:

83.168 Nm

Explanation:

I=mass of bat* ( length )^2 /3

Moment of inertia from one end = 2.2 * (0.96)^2 / 3.

Ang. acc. = 2.2×2 pi / 0.2 = 22pi

Torque =  2.3 * (0.96)^2 / 3 * 22 pi = 83.168 Nm SI units

he mass of the moon is 1/81 of the mass of the earth. Compared to the gravitational force that the earth exerts on the moon, the gravitational force that the moon exerts on the earth is

Answers

Answer:

It is the same in magnitude and opposite in direction.

Explanation:

According Newton's Universal Law of Gravitation, the force that any mass exerts on any other one, is proportional to the product of their masses, inversely proportional tho the square of the distance between their centers, and acts along the line that joins both points.At the same time, this force is part of a action-reaction pair, according to Newton's 3rd Law, that states that for a force that one body exerts upon another, there exists an equal and opposite force exerted by the other body on the former one.So, we conclude that the gravitational force that the earth exerts on the moon is equal in magnitude and opposite in direction to the one that the moon exerts on the earth.

The gravitational force the moon and the earth exert on each other are equal.

According to Newton's third law of motion, action and reaction are equal and opposite.

The gravitational force the earth exerts on the moon is equal in magnitude but opposite in direction to the gravitational force the moon exerts on the earth.

The magnitude of the gravitational force between the moon and the earth is calculated as follows;

[tex]F = \frac{GM_e M_m}{R^2}[/tex]

Thus, the gravitational force the moon and the earth exert on each other are equal.

Learn more about  gravitational force here: https://brainly.com/question/17301500

who win tug of war with what net force

Answers

Answer:

The force on rope is equal for both of them at any time.

For winning the game the force on ground is responsible.

Explanation:

Hi so this is 15 points :D
The layers of Earth are the crust, mantle, and core, with the core being divided into inner and outer layers. Which of the following does NOT describe the layers of the solid Earth?

(A) The core makes up the majority of Earth's volume.

(B) The mantle is composed of rocks known as silicates.

(C) The crust is extremely thin when compared to the core or mantle.

(D) The core is made up of dense elements, such as iron and nickel.

Answers

Answer:

A. The core makes up the majority of Earth's volume.

Explanation:

Although the core and mantle are about equal in thickness, the core actually forms only 15% of earth's volume whereas the mantle takes up 84%

ball a 0.604 kg moving right at 11.6 m/s makes a head-on collision with ball B at rest. after, ball A moves right at 2.09 m/s, and ball B moves right at 5.03 m/s. what is the mass of ball B? unite=kg

Answers

Answer:

1.142

Explanation:

A standing wave with a fundamental frequency 1.25x10 2 Hz is set up on a string of length 0.750 meters. What is the frequency of the third harmonic?

Answers

Answer:

500Hertz

Explanation:

Given the fundamental frequency of the string given as  1.25x10²Hz

Harmonics are integral multiples of the fundamental frequency.

In strings, the third harmonic F3 is expressed in terms of the fundamental frequency expressed as;

F3 = 4F0

F0 is the fundamental frequency;

Given

F0 = 1.25x10²Hz

F3 = 4(1.25x10²)

F3 = 4(125)

F3 = 500Hz

Hence the  frequency of the third harmonic is 500Hertz

What would be harder to accelerate- a 4kg shot put ball or a 2,000 kg boulder

Answers

Answer:

2,000 kg boulder

Explanation:t

The Inertial is the resistance of a body at rest to be in motion

Given the following  item

1. 4kg shot put ball  

2. 2,000 kg boulder

The body of greater mass has greater inertia, hence would be harder to accelerate.

An object with a small inertial mass will accelerate more than an object with a large inertial mass when acted upon by the same force.

Of the 2 items in the list, the 2,000 kg boulder has the largest mass, hence it also has the greater inertial  and more difficult to accelerate.

Which of following is true statement of self-efficiency

Answers

Explanation:

Which of the following best defines self-efficacy? The belief in one's ability to cause an intended event to happen. ... If Bobby does not value athletic skill, his lack of talent will have less impact on his self-esteem than if he highly values athleticism.

What is the period of a pendulum near Earth’s surface that is 130 cm long?
Question 1 options:

2.3 s

0.83 s

9.1 s

22.9 s

Answers

Answer:

A). 2.3 s

Explanation:

Given that,

Length of the string = 130cm or 1.3 m

g near Earth's surface = 9.8 [tex]ms^{-2}[/tex]

To determine the period of a pendulum near Earth’s surface;

T = 2 π [tex]\sqrt{\frac{L}{g} }[/tex]

T = 2π[tex]\sqrt{\frac{1.3}{9.8} }[/tex]

T = 2.3 sec

Therefore, the period of the pendulum near the earth's surface is 2.3 sec.

i need help with the one someone?

Answers

Answer:A road sign gives drivers informaton about traffic on busy highways. One sign shows 15 miles in 20 minutes. What is this speed in miles per hour?

Explanation:

5.
The diagram below shows two forces applied to a 2.0-kilogram block on a frictionless, horizontal surface.
F = 8.0 N
2.0 kg
F2 = 3.0 N
Frictionless Surface
What is the acceleration of the block?
A 4.0 m/s2 to the left
B.O 2.5 m/s2 to the right
C.O 2.5 m/s2 to the left
D. 1.5 m/s2 to the right

Answers

Answer:2.5m/s^2 to the left

Explanation:

The acceleration of the block is "2.5 m/s² to the left"

Given,

Force,

[tex]F_1 = 8 N[/tex][tex]F_2 = 3 \ N[/tex]

Mass,

[tex]m = 2 \ kg[/tex]

Considering equilibrium of forces on block,

then,

→ [tex]F_1 -F_2 = ma[/tex]

By substituting the values, we get

→     [tex]8-3 = 2a[/tex]

→           [tex]5 = 2a[/tex]

            [tex]a = \frac{5}{2}[/tex]

            [tex]a = 2.5 \ m/s^2[/tex] (to the left)

Thus the above answer i.e., option C is right.

Learn more:

https://brainly.com/question/21517582

An 8.00- W resistor is dissipating 100 watts. What are the current through it and the difference of potential across it?

Answers

Answer:

I= 3.5 amps

Explanation:

Step one:

given data

rating of resistor R= 8 ohms

power P= 100W

Required

The current I

Step two

Yet this power is also given by

[tex]P = I^2R[/tex]

make I subject of the formula we have

[tex]I= \sqrt{\frac{P}{R} }[/tex]

substitute

[tex]I= \sqrt{\frac{100}{8} }\\\\I=\sqrt{12.5}\\\\I= 3.5 amps[/tex]


Martin needs to push a box up a ramp.
Which force will make it hard for him to push the box?
friction
inertia
magnetism

Answers

Uh I think it is inertia but I could be wrong

When we walk, our legs alternatively swing forward about the hip joint as a pivot. In this motion the leg is acting approximately as a physical pendulum. Treating the leg as a thin uniform rod of length 0.80 m, find the time it takes for the leg to swing forward.

Answers

Answer:

0.3 seconds

Explanation:

When we walk, our legs alternatively swing forward about the hip joint as a pivot. In this motion the leg is acting approximately as a physical pendulum. Treating the leg as a thin uniform rod of length 0.80 m,

To find the time it takes for the leg to swing forward, where g = 9.8 m/s^2

Let's use the simple pendulum formula which says that:

T = 2π( sqrt ( L/g))

T = 2π[sqrt (0.8/9.8)]

T = 2π × 0.082

T = 0.5129

For the leg to swing forward only, the number of complete oscillations will be half.

Period T = time t ÷ number of complete oscillations. That is,

T = t ÷ 1/2

0.5129 = t ÷ 1/2

t = 0.5129 × 1/2

Time t = 0.256 seconds.

Therefore, the time it takes for the leg to swing forward is 0.3 seconds approximately

A neutron star consists of neutrons at approximately nuclear density. Estimate, for a 10-km-diameter neutron star its mass number, its mass, the acceleration of gravity at its surface.

Answers

Answer:

a) 7.2*10^55

b) 1.2*10^29 kg

c) 3.2*10^11 m/s

Explanation:

Given a diameter of 10 km, that's a radius of 5 km.

We first attempt to find its mass number

A = (r/1.2*10^-15)³

A = (5/1.5*10^-15)³

A = 7.2*10^55

This gotten mass number is then multiplied by the mass of each neutron in the star. With each neutron being, 1.7*10^-27

M = 7.5*10^55 * 1.7*10^-27

M = 1.2*10^29 kg

And finally, using the formula

g = GM/r², we can find its acceleration due to gravity.

g = (6.67*10^-11 * 1.2*10^29) / 5000²

g = 8*10^18 / 2.5*10^7

g = 3.2*10^11 m/s

the maximum value of magnetic in an electric field 3.2 *10^4​

Answers

Answer:

the answer is 12 because if your magnetic value and Electric field is 3.2 the answer will be 12

what is the maximum powr of a module in Watts to the nearest whole Watt?​

Answers

Complete question is;

You are looking at a module specification

sheet that has the table of information

below. What is the maximum power of this

module in Watts to the nearest whole Watt?

Value

Polycrystalline si

Characteristic

Cell Type

Cell

Configuration

Voc

160 in series

137.2 V

V_imp: 29.3 V

Ilsc: 8.60 A

I_Imp: 8.02 A

Dimensions (mm/in): 1000 x 1600 x 50 mm / 39.4" x 63" x 2"

Weight: 10 kg / 22 lbs​

Answer:

P ≈ 235 Watts

Explanation:

Formula for power is;

P = IV

Now, for maximum power, we will make use of I_imp and V_imp given

Thus, P = I_imp × V_imp

We are given;

V_imp: 29.3 V

I_Imp: 8.02 A

Thus: P = 8.02 × 29.3 = 234.986 Watts

We are to approximate to the nearest whole watt.

Thus: P ≈ 235 Watts

Other Questions
You all should know that...you guys are amazingI just wanted you to know thatthat's ityou can stop readingwhy are you still reading?I said that was it so why are you readingstop readingI said stop!ok finewell if your gonna keep reading you might as well hear thisjk im not writing anymorethat was all it wasJK AGAIN!you are all... wonderfulsmartbeautiful (inside and out)lovedand cared forso don't goyou gotta be hereyou gotta be here to spread your amazingnessbecause you are a queen/king Complete the ratio table below A fair coin has two faces called 'heads' and 'tails'. In a single toss of a faircoin, the probability of obtaining a tail is *A) 0B) 0.25C) 0.5D) 1 HELP PLEASE!!!!!!!!!!! i need help. i have a picture of my problem below. Why was the founding of St. Petersburg important to Russia? a. It proved that the Russians could defeat Austria and Prussia. b. It showed that the army was the most important institution in Russia. c. It allowed for expansion of Russian territory. d. It showed that a woman could be as effective a monarch as a man. on a map with a scale 1: 25000,the area of a lake is 33.6 square centimeters. calculate the actual area of the lake, giving your answer in square kilometers How do you think the missionaries used religion for European interests? Need help !!!! Yoko spent a total of 35 at the grossers store of this amount she spent 14 on fruit what percent did she spend on fruit A cube has an edge length of 19in. What is its volume, in cubic in? Nuestro equipo de baloncesto no ____(perder) muchos partidos SOMEONE HELP ON NUMBER 1 Brianna answers 5 questions for each chapter of a book she reads in English class.Write an equation to represent the total number of questions, q,Brianna answers based on the number of chapters, C,in the book she reads.Enter your equation in the box. If you could have an all-expenses paid trip to see any famous world monument, which monument would you choose? Which set of ordered pairs does not represent a function?o{(-1,-9), (5,5),(-1,-6), (4,4)}o {(8, 7),(-6,-9), (4,6), (7,-9)}o {(5,7), (-4,1),(0,4), (-9,6)}o{(-5,1), (8,0), (0,7), (5,7)} what is the dilation of 1/4 name as many tv shows as you can!!!if u name more than 10 you get brainilest!! Help with number 1 please What is the value of k? What was the belief of those who opposed political parties? Those who supported them?